Which one of the following, if true, would best serve as supporting evidence for the author's explanation of the econ...

liwenong28 on January 5, 2021

Why is A wrong?

Can someone please explain? Thanks.

Replies
Create a free account to read and take part in forum discussions.

Already have an account? log in

shunhe on January 8, 2021

Hi @liwenong28,

Thanks for the question! (E) is wrong because of rule 1. Remember, at most one person leaves more than one message. But in (E), both H and L leave two messages. So rule 1 is violated, eliminating (E).

Hope this helps! Feel free to ask any other questions that you might have.

lklop on July 8, 2021

I'm not sure if this answer is meant for the question^